MBE--Evidence(Barbri Outline) Flashcards

1
Q

I. RELEVANCE

A. Basic Principles

A
  1. Evidence is RELEVANT if it has Any Tendency to make a fact of consequence more probable or less probable than would be the case without the evidence.

2 Components:
Materiality: “of consequence” (matters) to the case
Probativeness: Any tendency to make fact of consequence more/less likely

  1. All irrelevant evidence is INADMISSIBLE.
    • Not relevant=not admissible
  2. All relevant evidence is ADMISSIBLE, UNLESS
    (a) some specific exclusionary rule is applicable, or
    (b) the court makes a discretionary determination that the probative value of the evidence is substantially outweighed by PRAGMATIC CONSIDERATIONS:

danger of unfair prejudice: emotional issue
confusion of the issues: side issue
misleading the jury: giving evidence too much weight
undue delay:
waste of time
unduly cumulative

How well did you know this?
1
Not at all
2
3
4
5
Perfectly
2
Q

I. RELEVANCE

B. Similar Occurrences

A

B. Similar Occurrences

IN GENERAL, if evidence concerns SOME Time, event or person OTHER THAN THAT INVOLVED IN THE CASE AT HAND, the evidence is INADMISSIBLE.

Why? Probative value is usually outweighed by pragmatic considerations (e.g., weak relevance, danger of confusion, misleading the jury, time-consuming).
BUT some recurring situations have produced concrete rules that may permit admissibility.

EXCEPTIONS:

  1. Plaintiff’s accident history.

HYPO 1. Phil drove into a lamp post and sues the municipality in negligence, alleging that the placement of the post created a hazardous condition. Should the municipality be allowed to introduce evidence that Phil has frequently driven into other stationary objects (tree, bridge, brick wall)?

GENERALLY, plaintiff’s accident history is INADMISSIBLE because it shows nothing more than the fact that the plaintiff is accident-prone. (propensity)

BUT, plaintiff’s prior accidents ADMISSIBLE if cause of plaintiff’s damages (injuries) is in issue.
(ie where did injuries originate?)

Q-TIP: Always ask yourself—For what purpose is the evidence being offered?

  1. Similar Accidents Caused by Same Event or Condition.

Generally, other accidents involving defendant are inadmissible because they suggest nothing more than general character for carelessness. But other accidents involving the same instrumentality or condition, and occurring under substantially similar circumstances, may be admitted for 3 potential purposes:
Existence of dangerous condition
Causation
Prior Notice to Defendant

HYPO 2. Assume in Hypo 1 that several other vehicles had collided with the same lamp post that Phil ran into. Could Phil introduce those other accidents against the municipality?
Substantial similarity is also the rule governing the admissibility of EXPERIMENTS and TESTS.
• Yes, if the other accidents occurred in substantially similar circumstances (weather, lighting conditions)
• And if so, those other accidents are admissible to show
o That the lamp post is a dangerous condition
o Causation (placement of post was a cause of accident)
o Prior notice to defendant

  1. Intent in Issue.

Person’s prior conduct may provide inference of intent on later occasion.

HYPO 3. Marta sues Brewski Co. for sex discrimination, alleging that she was qualified for the job but was not hired because she is a woman. She seeks to show that Brewski hired no women, despite their qualifications, during the past 6 years. Admissible?
• Yes. P has to prove she was fired bc she was a woman. P has to prove intent to discriminate. Intent is an issue here so you can use evidence of prior treatment of women to raise an inference of discriminatory intent bc intent is an issue, the prior treatment of women is admissible.

  1. Comparable Sales on Issue of Value.

Selling price of other property of similar type, in same general location, and close in time to period at issue, is some evidence of value of property at issue.

  1. Habit.

Habit of a person (or routine of a business organization) is ADMISSIBLE as circumstantial evidence of how the person (or business) acted on the occasion at issue in the litigation.

DISTINGUISH: Character evidence refers to a particular person’s general disposition or propensity. Character is usually not admissible to prove conduct on a particular occasion. E.g., Fact that Carlos is a “careless” driver is inadmissible to suggest that he ran a red light and caused the accident involving the plaintiff.

FAKE HYPO. Bob is on trial for the crime of failing to brush his teeth on Tuesday, November 5, 2005. Bob’s wife testifies that she can’t remember whether Bob brushed his teeth on November 5, 2005 but she also testifies that 1) Bob’s morning routine involves brushing his teeth immediately after showering at their bathroom sink and that he has stuck to this routine since they were married five years ago and 2) Bob has a reputation of being very clean and hygienic.

HABIT has 2 defining characteristics:
Frequency of conduct
Particularity of circumstances

Thus, habit is a repetitive response to a particular set of circumstances.

HYPO 4. In an auto accident case, the issue is whether Joe Isuzu stopped his car at the stop sign at the intersection of Hickory and Main Streets.

(a) Plaintiff calls Wanda to testify that during the six months preceding the accident, she had seen Joe run red lights, change lanes without using signals and run stop signs throughout town. Admissible as habit evidence to prove that Joe ran the stop sign at Hickory and Main?
• Frequency met particularity NOT met=INADMISSIBLE

(b) Wanda will testify that she has seen Joe run the stop sign at Hickory and Main on at least 8 occasions within a two-week period. Admissible as habit?
• Frequency met; particularity met = ADMISSIBLE

KEY WORDS: “always,” “invariably,” automatically,” “instinctively.”

Business Routine: Example: To prove that a particular letter was mailed by CEO, evidence that CEO put letter in her out-box on Tuesday, and messenger “routinely” picks up mail in CEO’s out-box at 3:00 P.M. each business day for delivery to mail room.

  1. Industrial Custom as Standard of Care.

Evidence as to how others in the same trade or industry have acted in the recent past may be admitted as some evidence as to how a party in the instant litigation should have acted, i.e., as evidence of the APPROPRIATE STANDARD OF CARE.

Example: Plaintiff is injured when a blade spins off a lawn mower. In an action against the manufacturer, she may show that 80% of all other lawn mower manufacturers, during the relevant time period, had installed devices to prevent blade spin-off.
• Shows what manufacturer should have done.

How well did you know this?
1
Not at all
2
3
4
5
Perfectly
3
Q

I. RELEVANCE

C. Policy-Based Exclusions. (jury doesn’t get to hear relevant evidence for policy reasons)

A

C. Policy-Based Exclusions. (jury doesn’t get to hear relevant evidence for policy reasons)

  1. Liability Insurance.

INADMISSIBLE TO PROVE THE
PERSON’S FAULT OR ABSENCE OF FAULT. BUT ADMISSIBLE for other relevant purpose, such as proof of OWNERSHIP / CONTROL OF INSTRUMENTALITY OR LOCATION, IF CONTROVERTED (disputed), or for purpose of IMPEACHMENT (Bias) of a witness.

HYPO 5. Gump falls down a well on Trump’s property, contending that the well was impossible to see because of overgrown foliage. Trump denies that he was negligent and also defends, in the alternative, on the ground that he did not own the land in question.

(a) Should Gump be allowed to introduce evidence that Trump carried a homeowner’s liability insurance policy on the land?
a. Not admissible to show FAULT but is admissible to show HE OWNED THE LAND.

(b) Same case. Marla, a witness called by Trump, testifies that she had been on Trump’s property just prior to the accident and there was no foliage covering the well. May Gump show, during cross-examination of Marla, that she is a claims adjuster employed by the company that issued the homeowner’s policy to Trump?
a. Yes, is admissible to show Bias.

BUT generally not admissible to impeach through Prior Inconsistent Statement or Contradiction

A LIMITING INSTRUCTION should be given to the jury whenever evidence is admissible for one purpose but not for another. Judge should tell jury to consider the evidence only for the PERMISSIBLE purpose.
2. Subsequent Remedial Measures. (JARTS CEO not wanting to fix it bc of future lawsuits: if you fix it, we wont let them use that against you to prove our fault)

Post-accident repairs, design changes, policy changes.

INADMISSIBLE for the purpose of proving NEGLIGENCE, CULPABLE CONDUCT, PRODUCT DEFECT, NEED FOR WARNING. Policy: To encourage post-accident repairs, etc. to avoid future accidents.

BUT such evidence may be ADMISSIBLE for some other relevant purpose, such as proof of OWNERSHIP / CONTROL or FEASIBILITY OF SAFER CONDITION, IF EITHER IS CONTROVERTED.

HYPO 6. Penelope bought a cup of coffee at Dante’s Coffee Inferno and scalded her tongue because the coffee was too hot. She sues Dante’s in negligence. Dante’s denies that it was negligent.
(a) At trial, Penelope seeks to introduce evidence that after the accident, Dante’s installed new thermostats on its coffee-brewing equipment. Penelope contends that this conduct is an admission by Dante’s that better safety controls were feasible. Admissible?
a. No. subsequent remedial measure inadmissible to prove NEGLIGENCE, CULPABLE CONDUCT, PRODUCT DEFECT, NEED FOR WARNING
(b) Same case, except now assume that Penelope contends that Dante’s negligence consisted of the failure to place warnings on its coffee cups indicating that its coffee was too hot for human consumption. Dante’s defends, in part, on the ground that it was impossible to affix labels to its coffee cups. Penelope seeks to introduce evidence that after the accident, Dante’s began to use cups that were pre-printed with warnings. Admissible?
a. Yes, admissible bc now proving feasibility of safer condition and controverted.
Texas: Same as federal rule with one exception: In a products liability action, evidence of written notification of a product defect sent by a manufacturer to a purchaser is admissible to prove existence of the defect.

HYPO 7. Miguel sues Universal Motors Inc. for injuries suffered in an auto accident. Miguel claims that a defect in the brakes of the Universal car caused the accident. Defense: “No defect.” Miguel seeks to introduce evidence that after the accident: (1) Universal changed the design of the brakes on its cars, and (2) sent a defect notice to purchasers urging them to bring their vehicles to service dealers for brake replacement. Admissible to prove the existence of a defect in the brakes at the time of Miguel’s accident?

Under Federal rule: (1) and (2) both subsequent remedial measures. Therefore INADMISSIBLE

Under Texas rule: (1) changing design of brakes INADMISSIBLE bc it’s a subsequent remedial measure and it’s not a written notification of defect. (2) ADMISSIBLE to prove existence of a defect.

  1. Settlements.

Civil: Evidence of a SETTLEMENT (compromise) or OFFER TO SETTLE a DISPUTED CLAIM is INADMISSIBLE TO PROVE LIABILITY OR WEAKNESS OF A PARTY’S CASE.
Why do we have this rule? To encourage settlement.

ALSO, STATEMENTS OF FACT made in the course of settlement discussions are INADMISSIBLE.
Why? To encourage free and open discussion of possible settlement.

BUT evidence of settlement may be ADMISSIBLE for purpose of IMPEACHMENT of a witness on the ground of BIAS.

HYPO 8. Hans and Franz were simultaneously struck by a truck being driven by Arnold. Hans and Franz both filed suit against Arnold, each seeking $100,000. Arnold denied all allegations.

(a) Before trial, Hans settled with Arnold for $50,000. When Franz’s case went to trial, Franz sought to introduce the Hans-Arnold settlement as evidence that Arnold, in effect, acknowledged his fault. Admissible?
a. No. Inadmissible. Franz is clearly trying to establish OFFER TO SETTLE TO PROVE LIABILITY

(b) Before Franz’s case went to trial, Franz and Arnold met to discuss possible settlement. During the discussion, Franz said, “I’ll accept $50,000 in settlement. The fact that I was jay-walking may confuse the jury.” Arnold declined. At trial, should Arnold be allowed to introduce (1) Franz’s offer to settle and (2) Franz’s admission that he was jay-walking?
a. (1): INADMISSIBLE: Settlement offers cant be offered to prove weakness of party’e case.
b. (2): INADMISSIBLE: bc it was a statement of fact made during settlement discussions.

(c) At the trial of Franz’s case, Arnold called Hans as a witness and Hans testified to the effect that Arnold did not drive negligently. On cross-examination of Hans, should Franz be allowed to prove the Hans-Arnold settlement?
a. ADMISSIBLE to show Hans’ Bias.

NOTE: The exclusionary rule only applies if there is a CLAIM that is DISPUTED (at time of settlement discussion) either as to VALIDITY or AMOUNT OF DAMAGES (HYPO 9AND 10).

HYPO 9. A’s and B’s cars collided. B immediately ran up to A and said, “Look, I’ll settle with you for $100,000 if you don’t sue.” Should A be allowed to introduce B’s statement against him at a subsequent trial?
ADMISSIBLE, bc not yet a DISPUTED CLAIM. Must have DISPUTED CLAIM first.

HYPO 10. After A’s and B’s cars collided, A sent a letter to B saying, “The accident was all your fault. I demand that you pay my damages in the amount of $100,000.” (CLAIM her) B called A on the phone and said, “You’re right about the accident. It was all my fault and I owe you the full $100,000 you’re asking for. But you know how fickle juries can be. If you don’t accept $50,000 now, you’ll have to sue me to get anything.” Should A be allowed to introduce B’s statements against B at a subsequent TRIAL?
ADMISSIBLE. There is a claim but based on what B said, there is not a dispute as to VALIDITY OF THE CLAIM or THE AMOUNT OF DAMAGES.

Variation: What if B had said, “It was all my fault, but you didn’t suffer $100,000 in damages”?
Here EXCLUSIONARY RULE applies. The settlement offer is INADMISSIBLE. Only need one (validity or amt in dispute) for exclusionary rule to apply.

Criminal: The following are INADMISSIBLE:
Offer to plead guilty—cannot be used against the defendant in the pending criminal case or in subsequent civil litigation based on the same facts.
Withdrawn guilty plea—cannot be used against the defendant in the pending criminal case or in subsequent civil litigation based on the same facts.
Plea of nolo contendere (“no contest”)—cannot be used against the defendant in subsequent civil litigation based on the same facts.
Statements of fact made during any of the above plea discussions.

BUT, a plea of GUILTY (not withdrawn) is ADMISSIBLE in subsequent litigation based on the same facts under the rule of party admissions.

  1. Offer to Pay Hospital or Medical Expenses

Evidence that a party has paid or offered to pay an accident victim’s hospital or medical expenses is INADMISSIBLE to prove liability.
Why? To encourage charity.
No need to show a DISPUTED claim.

HYPO 11. Donna’s car hit pedestrian Pablo. Donna immediately ran to Pablo and said, (a) “Don’t worry about a thing. I’ll pay for your hospital bills. (b) I’m sorry I ran the red light.”

(a) Is statement (a) admissible against Donna?
a. INADMISSIBLE. Offer to pay.
(b) Is statement (b) admissible against Donna?
a. ADMISSIBLE.

How well did you know this?
1
Not at all
2
3
4
5
Perfectly
4
Q

I. RELEVANCE

D. Character Evidence (Heavily Tested on MBE 6-8 MCQs)[aka Propensity Evidence]

A

D. Character Evidence (Heavily Tested on MBE 6-8 MCQs)[aka Propensity Evidence]

Character evidence refers to a person’s general propensity or disposition, e.g., honesty, fairness, peacefulness, or violence.
Potential purposes for the admissibility of character evidence:
(1) Person’s character/trait is a MATERIAL ELEMENT IN THE CASE.
(2) Character evidence to prove CONDUCT IN CONFORMITY WITH CHARACTER at the time of the litigated event, a/k/a character as circumstantial evidence of conduct on a particular occasion.
(3) Witness’s bad character for truthfulness to IMPEACH CREDIBILITY.

  1. Character Evidence in Criminal Cases.
    (a) Defendant’s Character.(When a D’s Character CAN/CANNOT be introduced)

Overview: Evidence of the defendant’s character to prove conduct in conformity is NOT ADMISSIBLE DURING THE PROSECUTION’S CASE-IN-CHIEF.

However, DEFENDANT, during the defense, MAY INTRODUCE EVIDENCE OF A RELEVANT CHARACTER TRAIT (by reputation or opinion testimony of a character witness) to prove conduct in conformity, thereby OPENING THE DOOR TO REBUTTAL by the prosecution.

HYPO 12. Rambo is charged with murder. During its direct case, should the prosecution be allowed to introduce evidence that Rambo has been convicted three times for assault, has a bad reputation for violence and recently stampeded a herd of cattle?
• No. bc this is evidence of the D’s Character for violence.
Should the prosecution’s proposed evidence be admitted on the ground that defendant’s violent character is a material element of the crime with which Rambo is charged?
• A Character trait is NEVER an element of a crime (can be element of defense).

HYPO 13. During the defense, Rambo calls Trautman to the stand to testify: (1) “I’m familiar with Rambo’s reputation for peacefulness, and it is excellent. (2) I personally know Rambo, and in my opinion he is a peaceful person.” Admissible? For what purpose?
• It IS ADMISSIBLE. D’s character for PEACEFULNESS is ADMISSIBLE when D is presenting.

When character evidence is admissible through a character witness to prove conduct in conformity, the only proper methods are:

REPUTATION (1) ‘have you heard’
OPINION (2) ‘did you know’

However, NO SPECIFIC INSTANCES OF CONDUCT

Could Trautman properly testify: “I’ve seen Rambo turn the other cheek when assaulted by bullies; he’s the President of the local Pacifist Club.”
• No. No specific instances of conduct allowed.
Could Trautman properly testify, “Rambo’s reputation for bravery and honesty is excellent”?
• No. MUST be RELEVANT Character traits.
o If Crime of VIOLENCE: Peacefulness
o If Crime of DECEIT: Honesty

(b) Prosecution’s Rebuttal

IF defendant has “opened the door” by calling character witnesses, the prosecution may rebut:

(1) by cross-examining defendant’s character witnesses with “Have you heard” or “Did you know” questions about specific acts of the defendant that reflect adversely on the particular character trait that defendant has introduced (prosecution must have good faith basis for the question); purpose: to impeach character witness’ knowledge; and/or
(2) by calling its own reputation or opinion witnesses to contradict defendant’s witnesses.

HYPO 14. During the defense, Rambo called Trautman to testify to Rambo’s peaceful character.
(a) Could the prosecutor ask Trautman, on cross-examination, (i) “Have you heard that Rambo was arrested last year for assaulting Rocky?” (ii) “Did you know that Rambo shot Judge Dredd three years ago?”
• Yes, ‘have you heard’ question. Must refer to violence.
(b) If Trautman denies having heard or knowing of the arrests or bad acts mentioned by the prosecutor, may the prosecutor prove that they actually occurred?
• Yes, ‘did you know’ question.
Side Note: Although, we can’t show SPECIFIC ACTS OF CONDUCT, here, the evidence is being put forth to show that D shouldn’t be believed rather than SPECIFIC ACTS OF CONDUCT.
(c) Could the prosecutor properly ask Trautman, “Have you heard (or did you know) that Rambo cheated on his income taxes last year?”
• No, Wrong Character Trait. Must refer to violence, not deceit bc this was case about violent crime.

HYPO 15. Assume Rambo introduced Trautman’s favorable character testimony. After the defense rests, the prosecution calls Murdock to testify that he has known Rambo for twenty years, is familiar with Rambo’s reputation for peacefulness in the community, and that such reputation is bad. Rambo’s attorney objects on the ground that this is impermissible character evidence.
• Objection should be overruled. Rambo opened the door. Reputation evidence is allowed after Defense opened the door by presenting its own character evidence first.
HYPO 16. Assume that the only witness who testified during the defense was Rambo himself, and he testified only to the fact that he did not commit the murder. After the defense rests, the prosecution calls Murdock to testify that Rambo has a reputation for violence. Rambo’s attorney objects on the ground that this is impermissible character evidence.
• Objection should be sustained. Rambo didn’t open the door simply by testifying. No evidence of good character was presented by Defense when Rambo took the stand.

(c) Victim’s Character—Self-Defense Case

Criminal defendant may introduce evidence of victim’s violent character to prove victim’s conduct in conformity, i.e., as circumstantial evidence that the victim was the first aggressor.

Proper method: character witness may testify to victim’s reputation for violence and may give opinion.

Prosecution rebuttal: evidence of victim’s good character (with reputation or opinion). In addition, under Federal rules only, prosecution may prove defendant’s character for violence/bad character.
Under Texas rules ONLY, rebuttal is limited to evidence of victim’s good character.

HYPO 17. Defendant, Coach Bobby, has been charged with assault for throwing a chair at Tonya. Coach Bobby claims that Tonya started the fight and lunged at him with a knife. To prove that Tonya was the first aggressor, Bobby calls Nancy to testify:
(a) That she knows Tonya and that in her opinion, Tonya is a very violent woman.
• Admissible. Self-defense case so defendant can offer character evidence that victim was the first aggressor (through reputation or opinion evidence=here, it was opinion).
(b) That she (Nancy) had been the victim of a knife attack by Tonya a few years ago.
• INADMISSIBLE. This is a specific instance of conduct to show poor character for violence.
(c) What if Bobby offers evidence that, at the time of the altercation with Tonya, he was aware of her prior knife attack on Nancy?
• ADMISSIBLE bc the specific act is not going to show bad character for violence, it’s going to show Bobby’s state of mind at the time he had to choose what to do in the situation (did he act reasonable? Depends on the circumstances. This fact is relevant to Bobby’s state of mind).

Separate rule of relevance: If the defendant, at the time of the alleged self- defense, was aware of the victim’s violent reputation or prior specific acts of violence, such awareness may be proven to show the defendant’s state of mind— fear—to help prove that he acted reasonably in responding as he did to the victim’s aggression.
(d) Victim’s Character—Sexual Misconduct Case

Under Federal “rape shield law”, in both criminal and civil cases, where defendant is alleged to have engaged in sexual misconduct, the following evidence about the victim is ordinarily inadmissible:

1) opinion or reputation evidence about the victim’s sexual propensity, or
2) evidence of specific sexual behavior of the victim

Exceptions:

(1) specific sexual behavior of the victim to prove that someone other than the defendant was the source of semen or injury to the victim;
(2) victim’s sexual activity with the defendant if the defense of consent is asserted; or
(3) where exclusion would violate defendant’s right of due process.

NOTE: Texas rape shield law applies only in criminal cases.

  1. Civil Cases
    (a) Character evidence generally INADMISSIBLE to prove CONDUCT IN CONFORMITY.

HYPO 18. A sues B for automobile negligence.
(a) During the plaintiff’s case in chief, A seeks to offer evidence of B’s reputation for careless driving. Admissible?
• INADMISSIBLE. That’s character evidence.
(b) During the defense, B calls Witness to testify that in her opinion, B is a prudent and careful driver. Admissible?
• INADMISSIBLE. No character evidence in a civil case.

BUT In a Civil Case based on Criminal Conduct:
HYPO 19. Nicole’s estate sues OJ for wrongful death damages, alleging that OJ intentionally killed Nicole. During the defense, may OJ properly introduce evidence of his peaceful character?
• INADMISSIBLE under Federal; ADMISSIBLE under Texas.
Federal rule: No character evidence. A civil case is a civil case even if based on criminal conduct.
Texas civil rule: A civil D accused of conduct involving MORAL TURPITUDE may introduce evidence of his Good Character by reputation/opinion testimony.

Moral Turpitude is a crime involving “grave infringement” of community sentiment
Examples of MT = Crimes of Dishonesty, violence and sexual misconduct.(Prostitution, theft, swindling, false report of a crime, man assaulting a woman) (NOT MT: DWI, gambling, public intoxication, man assaulting a man)

HYPO 20. In the wrongful death action against OJ, he defends on the ground of self-defense. May OJ properly introduce evidence of Nicole’s violent character to prove that she was the first aggressor?

Federal rule: INADMISSIBLE, it’s a Civil case.
Texas civil rule: ADMISSIBLE. A civil D accused of ASSAULTIVE Conduct, may prove the victims violent character (by reputation/opinion evidence) to suggest the victim was first aggressor.

(b) Evidence of person’s character is ADMISSIBLE in civil action where such character is an ESSENTIAL ELEMENT OF A CLAIM OR DEFENSE (provable by reputation, opinion, and specific acts). Only 2 situations:

1) Negligent hiring or Entrustment Cases
2) Defamation Cases

-Here, the character trait is an element of the claim of the defense. The character trait of the person is a defense. You can use specific instances of conduct.

Example (1): P was struck in 2004 by a truck being driven by Charlie, who was acting within the scope of his employment for Acme Trucking. P sues Acme, alleging that Acme was negligent in hiring Charlie in 2003 and thereafter allowing him to drive on Acme’s behalf. (Tort theory: Acme knew or should have known that Charlie was an accident risk.)
o P may introduce the testimony of character witnesses that Charlie had a reputation for being a careless driver, and they have a low opinion of Charlie’s care in driving. P may also prove that Charlie had been involved in three prior accidents.
Example (2): P sues Newspaper for libel based on a story in which P was accused of being dishonest.
o To support its defense of “truth,” Newspaper may introduce reputation, opinion and specific-act evidence about P’s dishonesty; and P may use the same type of evidence to show P’s honesty.

How well did you know this?
1
Not at all
2
3
4
5
Perfectly
5
Q

I. RELEVANCE

E. Defendant’s Other Crimes for Non-Character Purpose.

A

E. Defendant’s Other Crimes for Non-Character Purpose.

General rule: Other crimes or specific bad acts of defendant are not admissible during the prosecution’s case-in-chief if the only purpose is to suggest that because of defendant’s bad character he is more likely to have committed the crime currently charged.

Example: D is charged with robbing bank A. The fact that D robbed bank B six months later would be inadmissible character evidence.

BUT, if defendant’s other crimes or bad acts show SOMETHING SPECIFIC ABOUT THE CRIME CHARGED—something more than mere bad character—such evidence may be ADMISSIBLE as evidence bearing on guilt.

Most common non-character purposes: “MIMIC”— 
MOTIVE 
INTENT 
MISTAKE OR ACCIDENT, ABSENCE OF 
IDENTITY 
COMMON SCHEME OR PLAN 

HYPO 21. Defendant is charged with the murder of Officer Garcia. The prosecution seeks to prove that Defendant was convicted and imprisoned five years ago for narcotics sales in the aftermath of an investigation and arrest made by Officer Garcia. Defendant objects on the ground of impermissible character evidence. What ruling?
• Overruled. Earlier crime not offered to show propensity. Evidence being showed to prove Motive—to get revenge.

HYPO 22. Defendant is charged with possession of narcotics with the intent to sell. He defends on the ground that he was merely a possessor and user—not a seller—of the drugs. The prosecution seeks to prove that Defendant sold drugs a year ago in the vicinity of the arrest in the current case. Admissible?
• ADMISSIBLE. Not showing Propensity, but showing Intent.

HYPO 23. Lizzie Borden is accused of intentionally killing her mother with an ax. Defense: accident. Prosecution seeks to show that Lizzie threw a knife at her mother during a family quarrel one week before the mother’s demise. The evidence:

(A) Is admissible because it shows Lizzie’s propensity for violence.

(B) Is admissible because it shows the ax incident was not an accident. Accident, Absence of

HYPO 24. D is charged with the armed robbery of a Wal-Mart in Austin early in the afternoon of July 1. Defense: mistaken identity. Prosecution seeks to introduce evidence that around noon on July 1, D robbed a Penneys and a Sears in Austin, in the same vicinity as the Wal-Mart.
• ADMISSIBLE to show Identity.

HYPO 25. Defendant is prosecuted for robbing the First National Bank. Defense: alibi. Prosecution introduces evidence that the robber wore a red ski mask, carried a .38 caliber gun and used a uniquely worded stick-up note. Prosecution then seeks to prove that Defendant used the same modus operandi when robbing the Second National Bank a year ago.
• ADMISSIBLE to show Identity.

HYPO 26. Defendant is charged with robbing the First National Bank. The prosecution seeks to prove that two days before the robbery, the Defendant stole a white Acura from a neighbor in the same town. The robber of the First National Bank used a white Acura for the “getaway.”
• ADMISSIBLE using Common scheme/plan for showing Identity.

Method of proof of MIMIC-purpose crimes:

  • by conviction, or
  • by evidence (witnesses, etc.) that proves the crime occurred: conditional relevancy standard—prosecution need only produce sufficient evidence from which a reasonable juror could conclude that defendant committed the other crime.
  • Upon defendant’s request, prosecution must give pretrial notice of intent to introduce MIMIC evidence. In all cases, court must also weigh probative value vs. prejudice and give limiting instructions if MIMIC evidence is admitted.
  • If relevant, MIMIC evidence can also be used in civil cases, such as tort actions for fraud or assault.

Q-TIP: Before using MIMIC Evidence, Court must insure that defendant is actually contesting the issue to which MIMIC evidence is addressed (e.g., identity, intent). If a MIMIC category is satisfied, prosecution may use other-crimes evidence as part of its case-in-chief; MIMIC evidence is not dependent on defendant’s introduction of favorable character evidence.
Example: In a murder case where Ds defense is Self-defense. Prosecution offers other crime to show it was really him for Identity purposes.
• IMPERMISSBLE because here Identity is not at issue. Defendant has to actually be contesting the issue which the MIMIC evidence goes to.

F. Other Sexual Misconduct to Show Propensity in Sex-Crime Prosecution or Civil Action—Federal Rules Only. (All the Propensity limitations above are about to go out the window bc case involves a sex crime)

UNDER FRE ONLY, in a case alleging sexual assault or child molestation, PRIOR SPECIFIC SEXUAL MISCONDUCT of the defendant is ADMISSIBLE as part of the case-in-chief of the prosecution (in a criminal case) or of the plaintiff (in a civil action) for any relevant purpose, including DEFENDANT’S PROPENSITY FOR SEX CRIMES, i.e., conduct in conformity with character.

How well did you know this?
1
Not at all
2
3
4
5
Perfectly
6
Q

II. AUTHENTICATION OF WRITINGS

A

Q-TIP: Whenever a writing appears on the exam, be alert to 3 potential issues (aside from relevance): authentication, best evidence rule, and hearsay.

OVERVIEW: A showing must be made that the writing is authentic (genuine), i.e., that it is what it purports to be. This is the process of authentication.

In the absence of a stipulation as to authenticity, a FOUNDATION must be made in order for the document to be admissible.

How well did you know this?
1
Not at all
2
3
4
5
Perfectly
7
Q

II. AUTHENTICATION OF WRITINGS

A. General Principles

A

A. General Principles

  1. BER = Misnomer. Better described as the “original writings” rule.
  2. BER = In order to prove the contents of a writing, recording, or photograph, an original must be produced.
  3. Key Inquiries
  • What does it mean “to prove the contents” of a writing
  • To what evidence does BER apply?
  • What is an “original?”
  • What are the exceptions to the BER?

HYPO 29. Bubba ordered 100 pounds of shrimp from Gulf Shrimp Co. pursuant to a written purchase order. In his suit for breach of contract, Bubba takes the stand and testifies, “I didn’t get what I ordered. The purchase order called for 3” jumbo shrimp and they delivered 1” mini-shrimp.” Which of the following would be a valid objection to Bubba’s testimony?
(A) The actual shrimp are the best evidence of what was delivered.
• Bad Objection. Shrimp are not writing, recording, or photograph.
(B) The purchase order is the best evidence of what the contract required.
• Good Objection.
Definition: A party who seeks to prove the contents of a writing (includes sound recordings, X-rays, films), must either produce the original writing or provide an acceptable excuse for its absence. If court finds excuse is acceptable, the party may then use secondary evidence—oral testimony or a copy.

How well did you know this?
1
Not at all
2
3
4
5
Perfectly
8
Q

II. AUTHENTICATION OF WRITINGS

B. When best evidence rule applies: When a party is seeking to prove the contents of a writing.

A

B. When best evidence rule applies: When a party is seeking to prove the contents of a writing.

Two principal situations:

  1. The writing is a legally operative document, i.e., the writing itself creates (not just evidence of) rights and obligations. Examples: patent, deed, mortgage, divorce decree, written contract.
  2. Witness is testifying to facts that she learned solely from reading about them (or seeing them) in a writing.

HYPO 30. Barney the Burglar is charged with breaking into a warehouse. No one witnessed the break-in, but it was captured on film by an unmanned surveillance camera. Officer Sipowitz testifies that he watched the film and it clearly shows Barney was the burglar. Objectionable?
• This is a ‘writing’ (video recording, film, etc.). Witness is testifying to event that he saw on video. Witness is Proving the contents of the writing. We have the original. So produce the film.

How well did you know this?
1
Not at all
2
3
4
5
Perfectly
9
Q

II. AUTHENTICATION OF WRITINGS

C. When best evidence rule does not apply [not proving contents of a writing]:

A

C. When best evidence rule does not apply [not proving contents of a writing]: When a witness with personal knowledge testifies to a fact that exists independently of a writing which records the fact (Testifying to something they have personal knowledge about but it happens that there’s a writing that records that fact).

HYPO 31. Alger Hiss is charged with committing perjury during his testimony at a congressional hearing. At trial, a congressional aide offers to testify to what Hiss said during the hearing. True or False: The aide’s testimony is improper because the transcript is the best evidence of what Hiss said.
• False, the aide has personal knowledge of what was said bc he heard it. He’s proving the contents of what he heard. BER is not really monitoring for the ‘best evidence’. BER says If your proving the contents of a writing, you have to provide the writing. Here, witness is NOT proving the contents of a writing. Witness is proving the contents of their memory of what he perceived
.
HYPO 32. Worker sues Boss for nonpayment of wages and failure to reimburse for expenses.

(a) Without producing any documents, Worker testifies, “I worked 100 hours and my expenses were $1,000.” Boss objects—“Best evidence rule. Produce the time sheets and expense receipts.”
• Objection should be overruled. Time sheets and expense receipts are not legally operative documents. They don’t create rights and obligations they just create record information. And the worker is not testifying to something he only knows bc he read them in a writing. How does he know how many hours he worked? He worked those hours. How does he know his expenses? He spent them.
(b) Without producing any documents, Boss testifies: “Worker’s time sheets show she worked only 80 hours, and the receipts show only $500 in expenses.”
• Boss is testifying to something he only knows bc he read them in a writing. Produce those original writings (time sheets and expense receipts).

How well did you know this?
1
Not at all
2
3
4
5
Perfectly
10
Q

II. AUTHENTICATION OF WRITINGS

D. What qualifies as the “original writing”?

A

D. What qualifies as the “original writing”?

  1. the writing itself; any counterpart intended to have the same effect; any negative of film or print from the negative; computer print-out.
  2. duplicate—any counterpart produced by any mechanical means that accurately reproduced the original (e.g., photocopy, carbon copy). Rule on duplicates: duplicate is admissible to same extent as original UNLESS it would be unfair (e.g., photocopy of fuzzy fax), or genuine question is raised as to authenticity of original.
  3. handwritten copy is neither an original nor duplicate
How well did you know this?
1
Not at all
2
3
4
5
Perfectly
11
Q

II. AUTHENTICATION OF WRITINGS

E. Excuses for non-production of original

A

E. Excuses for non-production of original

  1. lost or cannot be found with due diligence
  2. destroyed without bad faith
  3. cannot be obtained with legal process

Court must be persuaded by preponderance of the evidence that excuse has been established; secondary evidence is then admissible (e.g., testimony based on memory, handwritten copy)

How well did you know this?
1
Not at all
2
3
4
5
Perfectly
12
Q

II. AUTHENTICATION OF WRITINGS

F. “Escapes” (not labeled as exceptions to BER but function as exception to the BER)

A

F. “Escapes” (not labeled as exceptions to BER but function as exception to the BER)

  1. VOLUMINOUS RECORDS can be presented through a summary or chart, provided the original records would be admissible and they are available for inspection.
  2. Certified copies of PUBLIC RECORDS
  3. COLLATERAL DOCUMENTS/WRITINGS (if not that important to lawsuit)

If court, in its discretion, determines writing is collateral, contents may be proven by secondary evidence.

How well did you know this?
1
Not at all
2
3
4
5
Perfectly
13
Q

IV. WITNESSES

A. Competency of Witness, In General

A

A. Competency of Witness, In General

  1. Basics: What has to be true of a witness before they give testimony? Testimonial Qualifications:

(a) Personal Knowledge
(b) Oath or Affirmation

  1. Juror as Witness (special rules where jurors can be witnesses)—juror may not testify:

a) in same case in which sitting as a juror as to ANY MATTER; or
b) in any other case as to STATEMENTS MADE IN DELIBERATIONS or
THE EFFECT ANYTHING HAD ON DELIBERATIONS.

BUT may testify as to any outside influence and extraneous prejudicial information

  1. Texas—in addition to personal knowledge and oath/affirmation:
    Witness incompetent to testify if court finds:
    -Insane at time of events witnessed or at trial, or
    -Child or other person lacks sufficient intellect to relate events witnessed
How well did you know this?
1
Not at all
2
3
4
5
Perfectly
14
Q

IV. WITNESSES

B. “Dead Man’s Statute” (Dead man’s rule in Tx)

A

B. “Dead Man’s Statute” (Dead man’s rule in Tx)

  1. In General (Multistate rules)
    (a) Witness is not ordinarily incompetent merely because she has an interest—a direct legal stake—in outcome of the litigation.
    (b) BUT under a typical state “Dead Man’s Act,” in a civil action, an interested party is incompetent to testify in support of her own interest against the estate of a decedent concerning communications or transactions between the interested party and the decedent.

HYPO 33. Shania sued Elvis for breach of an oral contract. Elvis denied that any contract was made. Elvis died before trial.

(a) May Shania testify to what Elvis said and did in negotiating the contract?
(b) May Shania’s friend Faith, who witnessed the making of the contract, testify to what Elvis said and did?

• On these facts, with the question written this way, both (a) and (b) are admissible bc federal rules don’t contain a dead man’s rule

Under the FRE, there is no “dead man’s rule.” Thus, on Multistate exam, witnesses ordinarily are not incompetent on this ground. BUT, if question explicitly states that the particular jurisdiction in which the case arises has a “dead man’s statute,” apply the rule in B.1.(b) above.

• Applying Dead Man’s rule to same hypo: (a) Shania is disqualified from testifying as to what Elvis said and did bc she is testifying as to a transaction between the 2 of them, she has an interest in the outcome. (b) admissible bc Faith not an interested party—not testifying in support of own interest.

ONLY APPLY DEAD MAN’S STATUTE IF MBE TELLS YOU THAT THE JURSIDICTION HAS A DEAD MANS STATUTE.

  1. Texas Dead Man’s Rule

Interested witness is incompetent to testify if:

(a) civil action by or against decedent’s estate, or by or against decedent’s heirs or legal representatives; AND
(b) either party to action seeks to testify to oral statement made by decedent

BUT party may testify to decedent’s oral statement IF EITHER:

  • decedent’s oral statement is corroborated by other evidence, OR
  • incompetent party (i.e. estate, heirs, legal representatives) is called by adverse party to testify concerning decedent’s oral statement

HYPO 34. Shania sues the administrator of Elvis’s estate in Texas state court for injuries she suffered in an auto collision. Elvis died shortly after the accident. If no one else witnessed the accident, may Shania, over a dead man’s rule objection, testify that immediately after the accident: (a) Elvis smelled of alcohol as he approached her, and (b) Elvis said to her “It was all my fault?”
• (a) is admissible bc Texas dead mans statute only applies to ORAL statements—not odors or things dead man did.
• (b) inadmissible. Tx dead mans rule would bar her from giving this testimony.

What result in Hypo 34 if Faith (Shania’s friend) also witnessed the accident and is willing to testify for Shania that Elvis admitted his fault?
• Faith is not disqualified bc she doesn’t have stake in outcome of case. ALSO, now Shania can also testify bc Shania’s statement is now being CORROBORATED.

How well did you know this?
1
Not at all
2
3
4
5
Perfectly
15
Q

IV. WITNESSES

C. Leading Questions

A

C. Leading Questions

Form of question suggests the answer (e.g., “Isn’t it a fact that . . . .”; or unevenly balanced alternatives)

(a) GENERALLY NOT ALLOWED on DIRECT EXAMINATION of witness.
(b) Generally ALLOWED on CROSS-EXAMINATION of witness.
(c) BUT allowed on DIRECT EXAM as follows:

(1) PRELIMINARY MATTERS
(2) YOUTHFUL/FORGETFUL WITNESS
(3) HOSTILE WITNESS
(4) ADVERSE PARTY

How well did you know this?
1
Not at all
2
3
4
5
Perfectly
16
Q

IV. WITNESSES

D. Writings in Aid of Oral Testimony (2 situations)

A

D. Writings in Aid of Oral Testimony (2 situations)

  1. Refreshing Recollection
    (a) Basic rule: Witness may not read from prepared memorandum; must testify on basis of current recollection.
    (b) BUT if witness’s memory fails him, he may be shown a memorandum (or any other tangible item) to jog his memory.

HYPO 35. Homer Simpson’s house was burglarized two years ago, and several valuable items were stolen. Simpson sued his insurer for failing to pay the loss covered by his homeowner’s policy. While on the stand at trial, Homer has trouble remembering all of the stolen items. To refresh Homer’s recollection, his attorney shows him a copy of a list of the missing items that Homer prepared for the police the day after the burglary. Insurer objects on the ground of lack of authentication, best evidence rule and hearsay.

(a) What ruling?
• Overruled as to all three objections. No need for authentication bc writing is not entered into evidence. Not BER bc not proving contents of writing—proving contents of memory. Not hearsay bc not being proved to prove anything—much less proving the truth of the matter asserted.
• ANY writing may be used to refresh recollection.
(b) If Homer’s recollection is refreshed, may he then read the list into evidence?
• No. The evidence is the testimony—not the writing.
(c) Safeguards against abuse: adversary has right:
1) To inspect the memory-refresher;
2) To use it on cross-examination; and
3) Your adversary has the right to introduce it into evidence

  1. Past Recollection Recorded (Hearsay Exception)

HYPO 36. In Hypo 35, Homer looks at the list of stolen items he prepared for the police the day after the burglary. It fails to jog his memory, and he is still unable to testify on the basis of current recollection. At this point, Homer’s attorney seeks to read the list into evidence. Objection: hearsay.

Hearsay exception for past recollection recorded. Foundation for admissibility of contents of writing:

(1) showing writing to witness fails to jog memory
(2) witness had personal knowledge at former time
(3) writing was either made by witness, or adopted by witness
(4) making or adoption occurred when event was fresh in witness’s memory
(5) witness can vouch for accuracy of writing when made or adopted.

• Show all the above things, and you get to read the past recollection recorded into evidence.

HYPO 37. After laying foundation, Homer’s attorney seeks to introduce Homer’s memorandum into evidence as an exhibit.

(a) Proper?
• Not proper; you have to READ it into evidence. Don’t want jury giving it undue weight.
(b) May the insurer have the memorandum introduced as an exhibit?
• Yes. The same procedural safeguards applying to prior recollection recorded apply here as well.

How well did you know this?
1
Not at all
2
3
4
5
Perfectly
17
Q

IV. WITNESSES

E. Opinion Testimony

A

E. Opinion Testimony

  1. Lay Witness (opinion by a non-witness)

(a) Lay opinion admissible if:
(1) RATIONALLY BASED ON WITNESS’S PERCEPTION (personal knowledge), and

(2) HELPFUL to jury in deciding a fact

(b) Examples: 
drunk/sober 
speed of vehicle 
sane/insane 
emotions of another person 
handwriting 
  1. Expert Witness (must be qualified as an expert)

(a) Qualifications:
education
AND/OR experience

Look for:
Skill
Kowledge
Experience
Education
Training

Texas rule on expert qualifications in cases based on “health care liability claims” (medical malpractice):

Expert must be actually practicing same type of health care as that of defendant, either at time of testimony or at time claim arose. (“Practicing” includes teaching at medical school or other relevant institution.) EX: plastic surgeon cant be testifying to heart procedure.

(b) Proper Subject Matter

Scientific, technical or other specialized knowledge that will be HELPFUL to jury in deciding a fact

(c) Basis of Opinion

Expert must have opinion based on “reasonable degree of probability or reasonable certainty”

AND 3 permissible data sources:

(1) personal knowledge (e.g., treating physician)
(2) other evidence in the trial record (testimony by other witnesses, exhibits (medical reports, X-rays))—made known to expert by hypothetical question
(3) facts outside the record if of a type reasonably relied upon by experts in the particular field in forming opinions

HYPO 38. Dr. Seuss, a board-certified child psychiatrist, testifies, “In my opinion, within a reasonable degree of medical probability, Bartholomew Cubbins’ preoccupation with hats is a disabling psychosis. My opinion is based on (1) my own clinical interviews and tests of Bartholomew; (2) exhibits 1 and 2 in evidence (MRI test results, medical office records of Dr. Grinch); (3) interviews of Bartholomew’s friends Wump, Gump and Thump; and (4) a written report prepared by Dr. Sam I. Am.

(a) Bartholomew moves to strike Dr. Seuss’s opinion because it is based, in part, on inadmissible hearsay.

(b) Should Dr. Seuss be permitted to testify further, “Let me read to you what Wump said during our interview . . . . . . . and here’s what was in Dr. Sam I. Am’s report”?
• NOT PERMISSIBLE: Expert is doing more than simply forming an opinion about the facts—he’s trying to communicate the facts to jury. Expert can say he relied on the information but can’t communicate the CONTENT of the information.

(d) Reliability

To be admissible, expert opinion MUST BE SUFFICIENTLY RELIABLE.

TX DISTINCTION: Federal: Court serves as “gatekeeper,” and will use four principal factors to determine reliability of principles and methodology used by expert (all types) to reach opinion (Daubert)—“TRAP” factors

Testing of principles or methodology
Rate of error
Acceptance by other experts in same discipline
Peer review and publication

Texas:
(1) If expert opinion is based on scientific methodology, court uses “TRAP ON” factors (Daubert/Robinson)— TRAP plus:

Objective vs. subjective interpretation of data
Non-judicial use of principle or methodology

Example: Toxicologist testifies about chemical cause of injury, based on laboratory studies.

(2) If expert opinion is based on non-scientific methodology, i.e., the expert relies on personal skill and experience, the test for reliability is less stringent. Court simply seeks to ensure that there is no analytical gap between expert’s methodology and the facts of the case.

Example: Engineer testifies about defect in seat belt, based on his many years’ experience testing safety of seat assemblies.

(e) Learned Treatise in Aid of Expert Testimony (Hearsay Exception)
(1) On direct examination of party’s own expert:

Relevant portions of treatise, periodical, or pamphlet may be read into evidence as substantive evidence (to prove truth of matter asserted) if established as reliable authority.

(2) On cross-examination of opponent’s expert:

Read into evidence to impeach and contradict opponent’s expert. Comes in as substantive evidence.

(3) BUT learned treatise may not be introduced as exhibit.
3. Ultimate Issues

Opinion testimony (lay or expert) is permissible even if it addresses an “ultimate issue” in the case (e.g., “X was drunk,” “insane”, “That’s X’s signature is on the check”). BUT—

HYPO 39. In a personal injury case, Defendant is alleged to have been driving recklessly at the time of a car accident. Witness who observed the event testifies that Defendant looked angry, smelled of alcohol and drove away from the scene at 80 m.p.h. Witness then states, “It looked to me as though Defendant was engaged in conduct constituting a reckless disregard for the safety of others.” Objectionable?

(A) Yes, because Witness is testifying to the ultimate issue.

(B) Yes, because Witness’s opinion is not helpful.

Texas: Expert witness may testify in terms of “negligence,” “proximate cause” or “lack of testamentary capacity” if proper legal standard is used.

Criminal Cases (FRE only): “Ultimate issue” is still proper objection if expert seeks to give direct opinion that defendant did or did not have relevant mental state (e.g., “D’s insanity prevented him from understanding that he was shooting at a human being.”)

How well did you know this?
1
Not at all
2
3
4
5
Perfectly
18
Q

IV. WITNESSES

F. Cross-Examination

A

F. Cross-Examination

  1. Party has a RIGHT to cross-examine any opposing witness who testifies at the trial. Significant impairment of this right will result, at minimum, in striking of witness’s testimony.
  2. Proper subject matter:
    (a) Matters WITHIN THE SCOPE of direct examination, and
    (b) Matters that test the witness’s CREDIBILITY
  3. Texas Distinction: Not limited to scope of direct exam; may question witness on anything relevant to case.
19
Q

IV. WITNESSES

G. Credibility and Impeachment, In General (6-8 Questions on MBE)

A

G. Credibility and Impeachment, In General (6-8 Questions on MBE)

  1. Bolstering Own Witness

IN GENERAL, not allowed until after witness’s credibility has been attacked.

HYPO 40. Plaintiff calls Witness 1 to the stand. Witness 1 testifies that she saw Defendant’s car run the red light. Defense counsel states that she has no questions for the witness. After Witness 1 steps down, Plaintiff calls Witness 2 who testifies, “Witness 1 has a good reputation for truthfulness.” Objectionable?
• Sustained. By calling 2nd character witness, that’s an attempt to bolster witness credibility. This is impermissible bolstering bc no attempt to discredit witness has been made by opposing counsel (defense).
o Cant strengthen witness credibility before it’s been attacked.

Variation: Witness 1, after testifying that she saw Defendant’s car run the red light, then testified, “I told everyone at work the next day that I had seen Defendant run the red light.” This is an inadmissible prior consistent statement(for 3 reasons):
• It’s bolstering
• It’s hearsay
• Has very minimal probative value (repetition of a lie doesn’t make it more true)

Exception: Prior identification of a person (e.g., Witness testifies that she recognizes D, sitting in court, as the perpetrator. In addition, “I picked D out of a line-up two weeks after the robbery”). Might seem like hearsay (out-of-court statement offered to prove truth of statement) but prior identification by trial witness is not barred by hearsay rule. It is labeled as “exclusion” from hearsay, and comes in as substantive evidence. Reliability factors: identification was closer in time to event, and witness on stand can be cross-examined.

  1. Impeachment of Own Witness

PERMITTED, without limitation.

20
Q

IV. WITNESSES

H. Impeachment Methods

A

H. Impeachment Methods

Overview:

(1) Prior Inconsistent Statements
(2) Bias, Interest or Motive to Misrepresent
(3) Sensory Deficiencies

Bad Character for Truthfulness:

(4) Bad Reputation or Opinion about witness’s character for truthfulness
(5) Criminal Convictions
(6) Bad Acts (without conviction) that reflect adversely on witness’s character for truthfulness (FRE Only)

(7) Contradiction

With respect to each impeachment method, consider 2 procedural issues:

(1) Can impeaching fact be proven by extrinsic evidence (documentary evidence or testimony from other witnesses), or is party bound by witness’s answers to impeaching questions?
(2) Assuming extrinsic evidence is permissible, must witness first be confronted with impeaching fact as a prerequisite to introduction of extrinsic evidence?
1. Prior Inconsistent Statements

Any witness may be impeached by showing that on some prior occasion, she made a material statement (orally or in writing) that is inconsistent with her trial testimony. Purpose: to suggest trial testimony is false or mistaken.

HYPO 41. Defendant is sued for negligence in a multi-vehicle accident in which he was driving his Suburban. Witness testifies for plaintiff that she saw the Suburban run the stop sign.

(a) On cross-examination, may Defendant’s counsel seek to establish that a few days after the accident, Witness told the police that the Jeep Cherokee, not the Suburban, ran the stop sign?
• Yes. It’s a prior inconsistent statement which can be used to impeach.
(b) If Witness admits she made the prior inconsistent statement, may Defendant use the statement as substantive evidence that the Jeep Cherokee, rather than the Suburban, ran the stop sign?
• No. a prior inconsistent statement, in general, is considered hearsay if offered to prove the matter asserted.
(c) BUT certain prior inconsistent statements may be admitted both for impeachment and as substantive evidence—to prove the truth of the matter asserted in the prior inconsistent statement: prior inconsistent statement given orally under oath AND as part of a formal hearing, proceeding, trial or deposition.

Example: Witness made her prior inconsistent statement about the Jeep Cherokee during a pretrial deposition in which she gave sworn testimony. This would be admissible to prove that the Jeep Cherokee, in fact, ran the stop sign.

Rationale: Prior inconsistent statements made under oath and during a formal hearing, etc., are reliable because of oath, and Witness is now subject to cross-examination about the statement.

Confrontation / Extrinsic Evidence Issue: Must Witness be confronted with her prior inconsistent statement while still on the stand, or may it be proven later by extrinsic evidence without such confrontation? I.e., at next opportunity for presenting affirmative evidence, call the person to whom she made the statement (if oral), or (if in writing) lay foundation to introduce writing into evidence.

Texas rule: Confrontation on stand usually required: Witness must be told the contents of the prior inconsistent statement, time and place, and person to whom made; and must be given an immediate opportunity to explain or deny the statement.

Federal rule: Confrontation timing is flexible: Not required to immediately confront Witness. But after proof by extrinsic evidence, Witness must be given an opportunity at some point to return to stand to explain or deny the prior inconsistent statement.

Exception: No confrontation required and no opportunity to explain need be given if Witness is the opposing party.

HYPO 42. In auto accident case, Plaintiff testifies that she was wearing her seat belt. Defendant does not cross-examine her. During the defense, Defendant calls Joe the Bartender, who testifies that Plaintiff told him, at Joe’s bar a week after the accident, that she had NOT been wearing her seat belt.

(a) Should Plaintiff’s motion to strike be granted on the ground that Plaintiff was not given an immediate opportunity to explain or deny the inconsistency?
• No on MBE (no opportunity to explain/deny to ANY witness); and
• No in TX (would be yes if .
(b) Is Plaintiff’s statement admissible to impeach Plaintiff AND as substantive evidence that she was not wearing her seat belt?

  1. Bias, Interest or Motive to Misrepresent

Examples: Witness is party; friend, relative or employee of party; expert witness being paid by party; person with grudge against a party, etc. Purpose: to suggest testimony is false, slanted, or mistaken in party’s favor.

Confrontation / Extrinsic Evidence Issue:

(a) Must witness be confronted with alleged bias while on the stand?

Texas rule: Yes. Witness must be told of circumstances or statements that allegedly show bias and given immediate opportunity to explain or deny.

Federal rule: Court’s discretion.

(b) If confrontation prerequisite is met, may bias be proven by extrinsic evidence?

Texas rule: Yes, if witness denies bias or fails to admit bias unequivocally

Federal rule: Yes, court has discretion to permit extrinsic evidence even if witness admits the bias.

  1. Sensory Deficiencies

Anything that could affect witness’s perception or memory. Examples: bad eyesight, bad hearing, mental retardation, consumption of alcohol or drugs at time of event or while on the witness stand. Purpose: to suggest mistake.

Confrontation required? NO
Extrinsic evidence allowed? YES

[4,5,6 below being used to show target witness has bad character for truthfulness]

  1. Bad Reputation or Opinion About Witness’s Character for Truthfulness

Any witness is subject to impeachment by this method.
Confrontation required? NO
Extrinsic evidence allowed? YES

Call a character witness to testify that Target Witness has bad reputation for truthfulness, or that character witness has low opinion of Target Witness’s character for truthfulness. Purpose: to suggest that Target Witness is not telling the truth on the witness stand.

HYPO 43. Larry testifies for the prosecution that he saw Defendant commit the crime. During the defense: Defendant calls Rev. Al to testify that Larry has a lousy reputation for truthfulness among members of Rev. Al’s congregation, and in Rev. Al’s opinion, Larry is not a truthful person.

(a) Admissible to suggest Larry’s testimony is false?
• Yes bc Larry testified.
(b) May Rev. Al follow up his opinion as follows: “Let me tell you how I reached my opinion of Larry. During the past year, he lied to me on six separate occasions.”
• No, no specific instances of conduct allowed—even to show basis of the opinion.

  1. Criminal Convictions

Purpose: to suggest testimony is false. Relevance: person who has been convicted of a crime is more likely to lie under oath than is a person with an unblemished record.

(a) Federal Rule:
(1) Conviction of any crime (felony or misdemeanor) involving dishonesty or false statement may be used to impeach any witness.
(2) If conviction does not involve dishonesty or false statement, it must be a felony, and court may exclude, in its discretion, if probative value on issue of witness credibility is outweighed by danger of unfair prejudice to a party (misuse as evidence of liability or guilt).
(3) Conviction, or release from prison, whichever is later, generally must be within 10 years of trial.

HYPO 44. Defendant is prosecuted for arson. At trial, Defendant testifies in his own behalf, urging that the fire was an accident. On cross-examination, may the prosecutor properly ask Defendant:

(a) whether he was convicted eight years ago for the misdemeanor of income tax fraud?
• Yes, bc he testified, but they have to be the right kind of convictions. This involves dishonesty and within 10yr time limit.
(b) whether he was released from prison nine years ago for his misdemeanor conviction for possession of marijuana?
• No. possession of marijuana is not a felony nor a crime of dishonesty/false statement.
(c) whether he was convicted two years ago for the misdemeanor of shoplifting?
• No, within 10 yrs, but is not a crime of dishonesty/false statement. (for purposes of MBE)
(d) whether he was convicted five years ago for felony assault?
• It’s admissible bc it’s a felony w/in 10yrs BUT SUBJECT TO court’s balancing test.

(4) Method of proof:

Ask witness to admit prior conviction, or

Introduce record of conviction (extrinsic). Not required to confront witness prior to introduction of record of conviction.

(b) Texas Rule

Same as Federal, with 3 distinctions:

  1. Types of conviction that can be used to impeach:

felonies of any type and crimes of moral turpitude.

  1. BALANCING of impeachment value against potential for prejudice applies to ALL convictions.
  2. Cannot use conviction to impeach if an appeal of the conviction pending.
    (c) Rehabilitation, Innocence, and Probation: Conviction is not Admissible if:
  3. Conviction was Subject of a pardon, annulment, certificate of rehabilitation, or “other equivalent procedure” based on a finding of rehabilitation. (both TX and MBE)

Unless:
Federal Rule: subsequently convicted of Crime punishable by death or imprisonment in excess of one year.

Texas Rule: subsequently convicted of felony or crime of moral turpitude.

  1. Was subject of pardon, annulment, or “other equivalent procedure” based on a finding of innocence.
  2. Texas Only: Successful completion of probation.
  3. Inquiry About Bad Acts (without conviction) if they reflect adversely on witness’s character for truthfulness

Confrontation on cross-examination is the only permissible means. No extrinsic evidence is permitted. Cross-examiner must have good-faith basis, and ability to inquire lies in court’s discretion.

HYPO 45. Witness gives favorable testimony for Defendant. On cross-examination, Plaintiff asks Witness whether she assaulted her mail carrier two years ago (no charges were ever brought). Objectionable?
• Yes, Objections should be sustained bc although a bad act, its not a bad act of deceit or lying.

HYPO 46. After Witness testifies for Defendant, Plaintiff asks Witness whether she made false statements in an application for food stamps in July 2001 (no charges were ever brought). Objectionable?

(a) Federal rule: Admissible bc bad act of falsehood, even though no conviction, still casts doubt on witnesses credibility
(b) Texas rule: Inadmissible. If bad act didn’t result in conviction, it is not important enough to waste time with.

HYPO 47. Same cross-examination (in federal court). Witness vehemently denies making false statements in the application for food stamps. May Plaintiff thereafter call a welfare agent to prove that Witness made the false statements?
• No. you have to take the witnesses answer. Inquiry on cross-examination is the only way you can prove it.

HYPO 48. Federal prosecution of Dieter. Hans testifies for Dieter. On cross-examination, Hans is asked whether he was “arrested” three years ago for passing counterfeit money. Objectionable?
• Yes, bc use of words like “arrested, charged, indicted.” Concern that jury will confuse these words with conviction.

HYPO 49. Prosecution of Donald. Winston testifies for the prosecution. On cross-examination, Winston is asked whether he was “arrested” a month ago for selling marijuana and is awaiting trial on those charges.

(a) Answer under Federal rule: Admissible, bc different reason for impeachment (testifying for prosecutions for quid ppro quo).
(b) Answer under Texas rule: Admissible.
7. Contradiction

Concept: Cross-examiner, through confrontation of witness, may try to obtain admission that she made a mistake or lied about any fact she testified to during direct examination. If the witness admits the mistake or lie, she has been impeached by contradiction. However, if she sticks to her story, the issue becomes whether extrinsic evidence may be introduced to prove the contradictory fact.

Rule: EXTRINSIC EVIDENCE NOT ALLOWED for the purpose of contradiction IF the fact at issue is COLLATERAL. A fact is collateral if it has no significant relevance to the case or to the witness’s credibility.

HYPO 50. In an auto accident case, Witness testifies for Plaintiff that, while leaning against a maple tree near the intersection of Yale and Harvard on March 1, he saw that the traffic light was red for Defendant as his car entered the intersection and hit Plaintiff. On cross-examination, Witness is asked (a) “Isn’t it a fact that the tree near the intersection of Yale and Harvard is an oak?” and (b) “Isn’t it a fact that the traffic light at the intersection of Yale and Harvard was not functioning at all on March 1?” Witness insists that his direct testimony was accurate.

(a) During the defense, may Defendant properly prove that the tree at Yale and Harvard is an oak tree?
• No. bc it’s a collateral fact, it’s not relevant to the case or on witnesses credibility. Witnesses just doesn’t know her trees very well.
(b) During the defense, may Defendant properly call a police officer to testify that the traffic light at the intersection of Yale and Harvard was not functioning at all on March 1?
• Yes. Not collateral. Color of traffic light is key to the case.

21
Q

IV. WITNESSES

I. Rehabilitation

A

I. Rehabilitation

  1. Showing witness’s good character for truthfulness.

(a) When?
• When witnesses credibility has been attacked on methods 4, 5 or 6 [from above].
o Bad reputation or opinion about witnesses character for truthfulness
o Criminal convictions
o Bad act that reflect negatively on witnesses character for truthfulness (without conviction).
• Then we can rehabilitate
(b) How?
• By calling own character witness that gives opinion/reputation testimony about target witnesses character for truthfulness.

  1. Prior consistent statement to rebut a charge of recent fabrication.

When? If the witness’s trial testimony is charged as a recent fabrication, or as a product of improper influence, a prior statement by the witness that is consistent with her testimony will be admissible to rebut the charge IF the statement was MADE BEFORE the motive to fabricate arose.

HYPO 51. Tom v. Nicole. On July 1, pedestrian Tom was struck by a car driven by Nicole. Penelope, a stranger to Tom and Nicole at the time, witnessed the accident and told the police on July 1 that Tom looked sober as he crossed the street. At trial, six months later, Penelope testifies for Tom, “He looked sober as he crossed the street.”

(a) On cross-examination, the only question Penelope is asked is whether she was convicted eight years ago of income tax evasion, to which she answers “Yes.” On re-direct, may Penelope properly testify that she told the police on July 1 that Tom had looked sober?
• Penelope’s character was attacked with a prior conviction (w/in 10yrs and dishonesty = shows poor character for truthfulness). How do we rehabilitate?
o Her credibility has been attacked by saying that she has a propensity to lie (tax evasion).
o We need to call a character witness to give reputation of Penelope in the community.
(b) Assume that on the cross-examination of Penelope, she is asked, “Isn’t it a fact that after this accident, you and Tom became close friends and are now living together as lovers?” to which she answers, “Yes.” On re-direct, may Penelope properly testify that she told the police on July 1 that Tom had looked sober?
• Penelope’s credibility is being attacked bc showing she has a motive to fabricate testimony (tom and her are lovers).
o This is where you use prior consistent statement to rebut a charge of recent fabrication. Made prior statement BEFORE motive to fabricate arose (at the scene), the motive to fabricate came AFTER that. ADMISSIBLE.

Penelope’s prior consistent statement, in this context, is admissible as substantive evidence that Tom had been sober, i.e., to prove the truth of the matter asserted in the statement.

22
Q

V. PRIVILEGES

A. Introduction

A

A. Introduction

  1. In general, on MULTISTATE exam, apply basic rules on privileges as covered in lecture.
  2. Federal procedure issue on MULTISTATE exam: If bar examiners specifically indicate the action is pending in federal court, apply the following procedural rules:
    (a) In federal-court action ARISING UNDER FEDERAL SUBSTANTIVE LAW (all civil cases arising under Constitution or federal statutes, and all criminal cases): “privileges are governed by the principles of the common law as they may be interpreted by the federal courts in the light of reason and experience.” For the most part, these are the basic rules on privileges as covered in lecture.
    (b) In federal-court action based on DIVERSITY jurisdiction, where state substantive law applies to parties’ claims and defenses (Erie situation), the federal court must apply PRIVILEGE LAW OF THE STATE whose substantive law is applicable. Note: In diversity actions, federal courts also apply STATE LAW ON COMPETENCY (e.g., Dead Man’s Statutes) and STATE LAW ON BURDENS OF PROOF AND PRESUMPTIONS. Aside from these 3 exceptions (privileges, competency, and burdens of proof / presumptions), FRE apply in all federal-court actions, including diversity cases.
  3. Texas: In general, apply basic rules on privileges as covered in lecture.
23
Q

V. PRIVILEGES

B. Attorney-Client Privilege

A

B. Attorney-Client Privilege

  1. Oldest common law privilege. Rationale: To encourage client to speak openly to counsel.
  2. Elements. Privilege applies to—
    Confidential communications
    between attorney and client (or representative of either)
    made during professional, legal consultation
    unless privilege is waived or an exception is applicable
  3. Definitions:
    (a) Confidential communications: Client must intend confidentiality (e.g., no privilege if client knows that third party is listening in; or if client asks attorney to disclose the communication to a third party). Joint client rule: If two or more clients with common interest consult the same attorney, their communications with counsel concerning the common interest are privileged as to third parties. But if the joint clients later have dispute with each other concerning the common interest, privilege does not apply as between them.

Communication: Privilege does not apply to underlying information, pre-existing documents, or physical evidence.

HYPO 52. Delbert is sued for his alleged negligence in an auto accident. He tells his attorney what happened and gives her the cell phone with which he was making a call at the time of the accident. Before trial, Delbert is deposed by plaintiff’s counsel:

(a) Must Delbert respond if asked, “What did you tell your attorney about the accident?”
o
(b) Must Delbert respond if asked, “Describe what you were doing at the time of the accident.”
o Yes. Privilege doesn’t cover the underlying facts.
(c) If served with a subpoena, must Delbert’s attorney produce Delbert’s cell phone?
o Yes. Privilege doesn’t protect device/physical evidence—just communications.

(b) Attorney—member of the bar or person that client reasonably believes is member of the bar

Representative of the attorney—any agent reasonably necessary to facilitate the provision of legal services (e.g., accountant working with attorney to “translate” client’s financial matters)

(c) Client—includes person seeking to become client (e.g., privilege attaches at outset of formal consultation with attorney even if client does not retain attorney)

Representative of client—any agent reasonably necessary to facilitate the provision of legal services (e.g., for corporate client, any employee who communicates with corporation’s attorney to enable attorney to provide legal services to the corporation)

(d) Professional legal consultation—primary purpose of communication must be to obtain or render legal services, not business or social advice
(e) Waiver—Client is holder of privilege, so client alone has power to waive by disclosure of communication to third party. Privilege continues after attorney-client relationship ends and even after death of client. Client’s estate representative has power to waive privilege after client’s death
(f) Exceptions—
(1) Future crime or fraud: E.g., Client tells attorney, “Help me disguise the bribes I made so that they look like legitimate business expenses.”
(2) Client puts legal advice in issue: E.g., In tax fraud prosecution, defendant defends on ground that she relied on advice of her attorney in reporting income.
(3) Attorney-client dispute: E.g., attorney sues client for unpaid fee, or client sues attorney for legal malpractice.

24
Q

V. PRIVILEGES

C. Physician-Patient Privilege

A

C. Physician-Patient Privilege

  1. Usually created by state statute. Rationale: to encourage candor by patient and to protect privacy.
  2. Elements. Privilege applies to—

Confidential communication or information acquired by physician from patient
for purpose of diagnosis or treatment of medical condition

Also applicable to psychotherapists (M.D. or other professional certified to diagnose or treat mental / emotional illness).

Federal distinction: in federal-court actions based solely on federal law, privilege exists for psychotherapists, but not for usual physician-patient confidences (e.g., gall bladder treatment)

HYPO 53. Physician examines Patient’s lungs in hospital room while visitor is present. (1) Patient tells doctor, “Do you suppose my wheezing is due to the 4 packs of cigarettes I smoke every day?” (2) After visitor leaves, Patient says to doctor, “Know any good lawyers? I haven’t paid my income taxes in 3 years.”

(a) In state court action in which condition of patient’s lungs is an issue, could doctor be compelled to disclose statement (1)?
• Yes. Bc it relates to diagnosis/treatment but a visitor was present so no confidentiality was intended.
(b) In prosecution for income tax evasion, could doctor be compelled to disclose statement (2)?
• Yes. Bc not related to medical treatment/diagnosis.

  1. General Exception: If patient expressly or impliedly puts physical or mental condition in issue. E.g., patient is plaintiff suing for damages for personal injury, or defendant asserts insanity defense.

Texas exceptions:

(1) no doctor-patient privilege in criminal cases. However, a communication to a person who is examining or treating another for drug or alcohol abuse is not admissible in a criminal proceeding.
(2) no privilege if any party relies on patient’s physical or mental condition as part of party’s claim or defense.

HYPO 54. Hillmon’s widow sues Big Bad Insurance Co. for failure to pay proceeds on Mr. Hillmon’s life insurance policy. Big Bad asserts that Hillmon misrepresented his heart condition when he applied for the policy. Big Bad subpoenas the medical records of Hillmon’s former physician showing Hillmon’s heart murmer. Privileged?
• Not in TX bc life insurance company has placed insured’s physical condition into issue.

25
Q

V. PRIVILEGES

D. Husband - Wife Privileges

A

D. Husband - Wife Privileges

  1. Spousal Immunity

IN CRIMINAL CASES ONLY, a spouse cannot be compelled to testify against the defendant spouse.

Rationale: to protect harmony of existing marriage at time of trial.

Witness-spouse, not defendant, is holder of privilege, i.e., witness-spouse may voluntarily testify against the defendant spouse if he/she so chooses.

  1. Confidential Communications Between Spouses

IN ANY TYPE OF CASE, a spouse is not required, and is not allowed in the absence of consent by the other spouse, to disclose a confidential communication made by one to the other during the marriage. Both spouses hold this privilege.

Rationale: to encourage candor between husbands and wives during the marriage? Privacy?

  1. Exceptions applicable to both privileges:

communications or acts in furtherance of future crime or fraud

communications or acts destructive of family unit, e.g., spousal or child abuse

additional Texas civil exceptions: all types of disputes between spouses (e.g., breach of contract) and incompetency / commitment proceedings

  • Spousal immunity: 1) married person whose spouse is the D in a criminal case may not be called as a witness by the P; and 2) married person may not be compelled to testify against spouse in any criminal proceeding.
    • Privilege belongs to witness-spouse. Privilege terminates on divorce
  • Confidential marital communications: either spouse, whether or not a party, has privilege to refuse to disclose, and to prevent another from disclosing, a confidential communication made between spouses while they were married.
  • Divorce doesn’t terminate this privilege retroactively. If communication is made in known presence of stranger, and spouse waives privilege as to herself (then cant refuse to disclose/prevent another from disclosing the communication), but the other spouse (one who didn’t reveal the communication) retains the privilege.

HYPO 55. Niles is prosecuted for the murder of his brother Frazier. Niles and Daphne are a married couple. Niles comes home on the night of Frazier’s demise wearing a blood-stained Armani topcoat, which Daphne observed.

(a) At trial, the prosecutor calls Daphne to the stand to testify to her observations about Niles’ topcoat, but she refuses to testify. The prosecutor seeks to compel her testimony.
• Daphne can’t be made to testify against husband as to anything.
(b) Assume Daphne is willing to testify against Niles. In addition to the topcoat observation, she seeks to testify to the following: “Niles told me when he got home that he stabbed Frazier.” Niles objects.
• Daphne can testify if she wants (as to observation of bloody topcoat) to and Defendant cant stop her, BUT Defendant can stop her from testifying as to confidential communications made (what Niles told her about stabbing Frasier)

HYPO 56. Assume that Daphne divorces Niles before his case goes to trial. The prosecutor calls her to the stand.

(a) Can Daphne be compelled to testify to her observations about Niles’s topcoat?
• Yes she can be compelled to testify. Spousal immunity doesn’t survive immunity.
(b) Can Niles prevent Daphne from disclosing his admission to her about stabbing Frazier?
• Yes. Can still prevent disclosure of confidential communications that occurred during marriage.

26
Q

VI. HEARSAY [Organization: 1) what is it/definition; 2) out of court statements that aren’t hearsay; 3) statements that meet definition BUT are EXCLUDED; 4) Exceptions to HS: statements that are HS BUT are ADMISSIBLE HS bc they meet an exception.]

A. 2-part definition

A

A. 2-part definition

(1) Out-of-court statement of a person (oral or written) AND
(2) Offered to prove the truth of the matter asserted in the statement

Hearsay rule: Hearsay is inadmissible unless an exception or exclusion applies.

So what’s wrong with hearsay? Inability of the opponent to cross-examine the Declarant (out-of-court speaker or author) at the time the statement was made. Declarant’s credibility is at issue (perception, memory and sincerity).

27
Q

VI. HEARSAY

B. Non-Hearsay Statements

A

B. Non-Hearsay Statements

Some out-of-court statements may look like hearsay at first glance, but are not hearsay if they are not offered to prove the truth of the matter asserted in the statement. An out-of-court statement may be relevant to some issue simply because it was spoken (or written). If offered for some other purpose, credibility of the declarant is irrelevant. On the issue of whether the statement was spoken, the witness on the stand can be cross-examined; or if the statement was in writing, it can be examined as an exhibit.

HYPO 57. Action by the estate of Percy against Damien seeking damages for the pain and suffering Percy experienced in an auto accident caused by Damien. Damien denies liability and also asserts that Percy died instantly in the accident. Witness on the stand proposes to testify that shortly after the accident, Percy said, “Damien’s car ran the red light.”

(a) Hearsay if offered to prove who ran the red light?
• Is HS. Yes. Being offered for its truth
(b) Hearsay if offered to prove that Percy was alive following the accident?
• Not HS if offered to prove Percy was alive following the accident.

28
Q

VI. HEARSAY

C. Principal Categories of Non-Hearsay Purposes

A

C. Principal Categories of Non-Hearsay Purposes

  1. Verbal Act (Legally Operative Facts/Words)

HYPO 58. Gates sued Trump for breach of an oral contract. Witness takes the stand and proposes to testify as follows: “I heard Trump say to Gates: “’I accept your offer to sell Microsoft.’” Hearsay?
• IT IS an out of court statement; BUT IT’S NOT being offered to prove the truth, it’s being offered to show Trump accepted the offer. Relevant Non-truth purpose here.

Similar: contract offer or cancellation, making gift, bribe, perjury, fraud, defamation, words accompanying ambiguous acts (e.g., D is charged with theft of X’s car; D testifies, “As X handed me the keys, he said I could have the car for the weekend.”).

  1. To Show Effect on Person Who Heard or Read the Statement

HYPO 59. Plaintiff v. Supermarket. Plaintiff alleges she slipped and fell on a broken jar of salsa in aisle 3 and that Supermarket had prior notice of the dangerous condition. Plaintiff’s witness takes stand and proposes to testify: “Several minutes before Plaintiff entered aisle 3, I heard another shopper tell Supermarket manager, ‘There’s a broken jar of salsa on the floor in aisle 3.’” Inadmissible hearsay?
• Depends on the proposition we’re offering it to prove. IT IS an out of court statement. The matter asserted in the statement is that there’s a broken jar of salsa on aisle 3. SO IT IS H/S if it’s being offered to prove that there really was a broken jar of salsa on aisle 3. BUT IT IS NOT H/S if it’s being offered to prove Supermarket had prior notice—this is a relevant non-truth purpose.

HYPO 60. Sybil is charged with the murder of her husband Basil. To prove motive, the prosecutor seeks to introduce an anonymous note to Sybil that was found in her possession at the time of her arrest. The note stated, “Basil is having an affair with Polly.” Inadmissible hearsay?
• IT IS an out of court statement; and IT IS NOT being offered to prove the matter asserted (that the affair was true or not) BUT rather it was being offered to show that Sybil read the note and this establishes motive. The out of court statement is being offered to prove motive. The statement serves a relevant non-truth purpose. Therefore, Not H/S and:
• ADMISSIBLE

  1. Circumstantial Evidence of Speaker’s State of Mind

HYPO 61. Homer is prosecuted for murder. Defense: Insanity. Witness for Homer proposes to testify: “Two days before the killing, Homer said, ‘I am Elvis Presley. It’s good to be back.’”
• IT IS an out of court statement. Is it being offered to prove matter asserted (if Homer really is Elvis)? NO. IT IS NOT being offered to prove the matter asserted. This is a relevant non-truth purpose. It is being offered to show circumstantial evidence that Homer was insane.

29
Q

VI. HEARSAY

D. Prior Statements of Trial Witness

A

D. Prior Statements of Trial Witness

HYPO 62. Prosecution of D for robbery. D takes the stand in his own defense and testifies: (a) “I didn’t do it. (b) And I told the cops when they arrested me that I didn’t do it.” Should (a) and (b) be excluded as hearsay?

General rule: A WITNESS’S OWN PRIOR STATEMENT, if offered to prove the truth of the matter asserted in the statement, is hearsay and is INADMISSIBLE unless an exception or exclusion applies.

3 Witness-Statement Exclusions from Hearsay (called “non-hearsay”):

  1. Witness’s prior statement of identification (cross-reference hypo 40)
  2. Witness’s prior inconsistent statement IF prior statement was under oath and made during formal trial, hearing, proceeding or deposition (cross-reference hypo 41)
  3. Prior consistent statement used to rebut charge of recent fabrication or improper motive or influence (cross-reference hypo 51b)
30
Q

VI. HEARSAY

E. Party Admissions (statements by opposing party) | Contrast w/H/S Exception Statement against interest

A

E. Party Admissions (statements by opposing party) | Contrast w/H/S Exception Statement against interest

  1. Any statement (even in court statements | contrast with former testimony exception) made by a party/representative w/authority to speak on behalf (plaintiff or defendant) is admissible against that party.
  2. Called “non-hearsay.”
  3. Theory: Party ought to bear the consequences of what she says. Can explain to jury, and cannot complain about inability to cross-examine self.

HYPO 63. X is charged with income tax evasion for the year 2000. Prosecutor wants to prove X’s income during 2000, and offers into evidence a loan application X submitted to a bank in that year. X objects on the ground that the loan application, which is filled with inflated numbers, was self-serving and unreliable.
• Any statement made by a party IS ADMISSIBLE against that party

HYPO 64. Ma v. Life Insurance Co. for non-payment of policy proceeds on the life of Pa. Defense: Suicide. Defendant offers a letter by Ma to her friend in which she wrote, “When I came home from shopping I found Pa dead on the floor with his revolver nearby. I didn’t see what happened, but this was no accident. Pa did himself in.” Admissible despite Ma’s lack of personal knowledge?
• Ma speculating that Pa killed himself. IT IS ADMISSIBLE even if lacked personal knowledge of what actually happened.

Vicarious Admissions

Statement by agent/employee is admissible against principal/employer if statement concerns matter within scope of agency/employment AND is made during agency/employment.

HYPO 65. Charlie the truck driver smashed into Pam’s house while on a run for Acme Trucking, his employer. Charlie descended from the cab and calmly told Pam, “Sorry about wrecking your home. I guess I took my eyes off the road. I was reaching down to get a beer and a joint.” In Pam v. Acme, is Charlie’s statement admissible against Acme?
• Yes. ADMISSIBLE against Acme bc statement about matter w/in scope of employment AND statement made during the employment.

HYPO 66. Betty v. Acme Trucking for sex discrimination in failing to hire her. She offers the statement of Charlie, an Acme truck driver, who told her over drinks one night, “I know the Acme personnel office has a policy against hiring women no matter how qualified they are.” Charlie’s statement is inadmissible because:

(A) Charlie was not on the job when he was speaking to Betty.

(B) Charlie’s statement did not concern a matter within the scope of his employment.

31
Q

VI. HEARSAY

F. Hearsay Exceptions

A

F. Hearsay Exceptions

Justified by reliability factors or other good reasons sufficient to excuse inability to cross-examine declarant:

  1. Former testimony (UNAV required) (Unavailability: criminal: incarceration in another state NOT enough | civil: incarceration is enough to prove unavailable)
  2. Statement against interest (UNAV required)
  3. Dying declaration (UNAV required)
  4. Excited utterance
  5. Present sense impression
  6. Present state of mind
  7. Declaration of intent
  8. Present physical condition
  9. Statement for purpose of medical treatment or diagnosis
  10. Business records
  11. Public records

(Also, recall Past Recollection Recorded, Learned Treatises)
PAST RECOLLECTION RECORDED
Under the Federal Rules of Evidence, a hearsay exception exists that allows a witness to read a writing to the jury if: (1) the witness once had personal knowledge of the writing; (2) if the witness now forgets the writing and showing the writing to the witness does not jog his or her memory; (3) the writing was either made by the witness or adopted by the witness; (4) the writing was made when the event was fresh in the witness’s memory; and (5) the witness can attest that, when made, the writing was accurate.

Criminal Defendant’s Right of Confrontation: “Testimonial Statements.” Regardless of whether a hearsay exception is satisfied, the sixth amendment right of confrontation prohibits the use of “testimonial” hearsay statements against a criminal defendant if the declarant is unavailable and the defendant has had no opportunity for cross-examination. “Testimonial” statements include sworn testimony:

(1) at a grand jury,
(2) prior trial, or
(3) preliminary hearing, and
(4) responses made during police questioning, whether sworn or unsworn.

Example: Police interrogate defendant’s accomplice at the station house and elicit incriminating statements. At defendant’s trial, the accomplice refuses to testify. The accomplice’s statement to the police, even if it falls within a hearsay exception (e.g., statement against interest), cannot be used against the defendant because it is testimonial and there has been no cross-examination.

Exception: “Declarant Unavailable Due to Wrongdoing.” Any type of hearsay statements are admissible against a defendant who engaged in or acquiesced in wrongdoing that was intended to, and did, procure the unavailability of the declarant as a trial witness. Theory: By making the witness unavailable through his own wrongdoing, defendant forfeits the sixth amendment objection.

Example: Tony is on trial for loan-sharking. Paulie made incriminating statements about Tony to the police and at a grand jury. Paulie was expected to be the key government witness at trial, but he has been found dead in the river. The court finds by a preponderance of the evidence that Tony is responsible for Paulie’s disappearance. Paulie’s grand jury testimony and interview statements to the police are admissible against Tony

32
Q

VI. HEARSAY

F. Hearsay Exceptions

  1. Former Testimony
A
  1. Former Testimony
    (a) The former testimony of a now-unavailable witness, if given at a former proceeding or in a deposition, is admissible against a party who, on the prior occasion, had an opportunity and motive to cross-examine or develop the testimony of the witness. Issue in both proceedings must be essentially the same.
    (b) Theory: reliability assured by cross-examination on prior occasion; however, we prefer live testimony, so witness must now be unavailable

HYPO 67. Bus accident. Passengers A and B were seriously injured. A sued Bus Co., alleging negligence by bus driver. At trial, Witness testified for A that bus driver was intoxicated at time of accident. Thereafter, Witness died. B now sues Bus Co. and seeks to admit a transcript of Witness’s former testimony.
• IT IS H/S, but is ADMISSIBLE under former testimony exception.

HYPO 68. Same bus accident. At grand jury, Witness testified that bus driver was intoxicated at time of accident. Thereafter, Witness died. Bus driver is prosecuted for DWI. Prosecutor seeks to admit a transcript of Witness’s grand jury testimony.
• H/S BUT does NOT meet former testimony exception bc party against who statement is being admitted was not afforded opportunity to cross-examine the witness. Therefore:
• INADMISSIBLE

(d) Grounds of Unavailability:
(1) death/serious illness
(2) absence from the jurisdiction
(3) privilege (spousal immunity/5th amendment)
(4) stubborn refusal
(5) lack of memory (eg. physically available but not for purposes of memory)

Note: Same grounds of unavailability apply to all exceptions where unavailability is a requirement—
Unavailability is a requirement for: former testimony, statement against interest, dying declaration.

Texas distinction: In civil actions, DEPOSITION of Witness taken in same proceeding is ADMISSIBLE without need to show that Witness has become unavailable.

33
Q

VI. HEARSAY

F. Hearsay Exceptions

  1. Statement Against Interest/Declaration against interest | Contrast with NON-H/S Party Admission (statement by opposing party)
A
  1. Statement Against Interest/Declaration against interest | Contrast with NON-H/S Party Admission (statement by opposing party)
    (a) An unavailable declarant’s statement against his or her

pecuniary
proprietary
or penal interest.

(b) Theory: Not likely to lie when making a personally damaging statement
(c) Statement against interest differs from party admission:

must be against interest when made
any person (not merely party) can make statement against interest
personal knowledge is required
declarant must be unavailable

HYPO 69. Plaintiff v. Acme Trucking, based on Charlie the truck driver’s negligent driving. Charlie was fired immediately after the accident. Two weeks later, Charlie told Plaintiff’s insurance adjuster that he had been drunk while driving. At trial, Charlie refused to testify on the ground of self-incrimination. The insurance adjuster may properly testify to Charlie’s statement as evidence against Acme because the statement is:

(A) A vicarious party admission.
• NOT APPLICABLE bc employment relationship not in existence at time statement was made
(B) A statement against interest
• Correct. Unavailable bc asserting the 5th.

(d) Qualification in criminal cases: Statement against penal interest, when offered to EXCULPATE defendant, must be corroborated

HYPO 70. Prosecution of Doppler for arson of Town Hall. Doppler calls Waldo to testify that while sitting in a bar, Waldo heard Stranger say, “I’m the guy who torched Town Hall, but I’m glad they think it’s Doppler. Just to be safe, I’m leaving town tomorrow.” Doppler’s attorney demonstrates that Stranger has not been located despite a diligent search.
• This meets statement against interest definition HOWEVER: we need corroboration.

(e) Texas distinctions for statement against interest:
(1) Unavailability not required
(2) Self-damaging statement includes one that makes declarant an object of hatred, ridicule or disgrace, i.e., against “social interest”.

HYPO 71. Elementary school principal sues Newspaper for libel for article accusing him of having sex with PTA mothers. To prove defense of truth, Newspaper calls Reporter to testify that Mothers A, B and C (all of whom are alive and well and live nearby) told him they had sex with the principal. Admissible—

(a) Under Federal rule?
• No. INADMISSIBLE bc: 1) the declarants are available. Also, 2) statements probably not against penal, pecuniary or proprietary interest.
(b) Under Texas rule?
• Yes. ADMISSIBLE bc: 1) tx doesn’t require unavailability and 2) these are the type of statements that would be against their ‘social interests’

34
Q

VI. HEARSAY

F. Hearsay Exceptions

  1. Dying Declaration
A
  1. Dying Declaration
    (a) Statement made under a belief of impending and certain death by a now-unavailable declarant concerning the cause or surrounding circumstances of the declarant’s death (mere suspicion not enough; must have some kind of actual knowledge of the circumstances).
    (b) Theory: No one wants to die with a lie on his lips.
    (c) Type of case:

Federal:
Criminal cases: HOMICIDE ONLY
Civil cases: ALL TYPES

Texas:
Criminal and Civil cases: ALL TYPES

HYPO 72. Prosecution of Dagger Dan for the murder of Victor Victim. A passerby found Victor lying in the gutter in a pool of blood with a knife in his stomach. Victor told the passerby, “It’s not looking too good for me. Dagger Dan did it, and I’m going to get him for this.” Victor died an hour later. May the passerby testify to Victor’s statement as a dying declaration?
• Probably not bc dying declaration must be made under belief of certain and impending death. Statement anticipates more life. Look for declarant to give up hope.

HYPO 73. Prosecution of Dillinger for bank robbery. At the scene, a bank officer spoke with wounded Teller Tim, who gasped, “I’m a dead man. Get me a priest. Dillinger shot me as he made his getaway.” Tim then lapsed into a coma from which he has not emerged. May Ness testify to Tim’s statement as a dying declaration—

(a) Under the Federal rule?
• No. this exception is only available for HOMICIDE cases in criminal prosecutions.
(b) Under the Texas rule?
• Yes. ADMISSIBLE. In a coma, under belief of certain/impending death, and statements concerned circumstances of declarants impending death.

HYPO 74. Same event except civil action against Dillinger for Tim’s personal injury damages. Tim is still in a coma. May Ness testify to Tim’s statement as a dying declaration—

(a) Under the Federal rule?
• Yes bc elements met.
(b) Under the Texas rule?
• Yes, same reasons.

“Spontaneous” Statements (Unavailability Not Required) [4-9 below]

35
Q

VI. HEARSAY

F. Hearsay Exceptions

  1. Excited Utterance
A
  1. Excited Utterance
    (a) Statement concerning a startling event and made while declarant is still under the stress of excitement caused by the event.
    (b) Theory: excitement suspends one’s capacity to fabricate.

HYPO 75. Ernie observes a horrific head-on auto collision and excitedly tells a cop, who arrives 10 minutes later, “Oh my God, Officer! Both of those cars were going 80 miles an hour!” May the cop properly testify to Ernie’s statement in subsequent litigation arising out of the accident?
• Yes, it’s an excited utterance.

36
Q

VI. HEARSAY

F. Hearsay Exceptions

  1. Present Sense Impression (could be mundane event; doesn’t have to be starling/exciting like Excitted utterance)
A
  1. Present Sense Impression (could be mundane event; doesn’t have to be starling/exciting like Excitted utterance)
    (a) Description of an event made while the event is occurring or immediately thereafter.
    (b) Theory: declarant has no time to fabricate

HYPO 76. Pedestrian alleges that Dora is the hit-and-run driver who struck him. Pedestrian testifies, “I saw a silver Acura speeding away. A couple of seconds later, some unknown bystander told me he saw the Acura and its license plate number was ‘007.’”
• ADMISSIBLE as present-sense impression. No time to fabricate.

37
Q

VI. HEARSAY

F. Hearsay Exceptions

  1. Present State of Mind (aka then-existing state of mind)
A
  1. Present State of Mind (aka then-existing state of mind)
    (a) Contemporaneous statement concerning declarant’s present state of mind, feelings, emotions.
    (b) Theory: contemporaneous statement about matter as to which declarant has unique knowledge.

HYPO 77. Probate of Wanda’s Will, in which she left all her money to the local pet cemetery. Wanda’s family challenges the will on the ground that Wanda was insane when she executed it. Pet cemetery offers testimony that a few days before execution of the will, Wanda said to her friend, “I do not love my family anymore.” Admissible over hearsay objection?
• Yes, ADMISSIBLE, statement of the-existing state of mind (what she’s feeling at the moment she speaks). Would NOT meet this exception IF Wanda had said “last month, I didn’t love my family anymore.”

38
Q

VI. HEARSAY

F. Hearsay Exceptions

  1. Declaration of Intent
A
  1. Declaration of Intent
    (a) Statement of declarant’s intent to do something in the future, including the intent to engage in conduct with another person
    (b) Theory: contemporaneous statement about matter as to which declarant has unique knowledge

HYPO 78. Susan has died and her family sues Life Insurance Co. for nonpayment of the policy proceeds. Defense: Suicide. Life Insurance Co. seeks to introduce a note found in Susan’s apartment (in Susan’s handwriting) in which she said, “I’m going to end it all next week.”
• ADMISSIBLE bc this was a Statement of a declarant’s intent to do a future act

HYPO 79. Prosecution of Raymond for murder of Vic. Before going out Monday night, Vic told wife, “I’m meeting Raymond tonight at the bowling alley.” Vic’s dead body was found Tuesday morning outside the bowling alley.
• ADMISSIBLE bc This is a statement of intent to do something in the future/engage in conduct with someone.

39
Q

VI. HEARSAY

F. Hearsay Exceptions

  1. Present Physical Condition
A
  1. Present Physical Condition
    (a) Statement made to anyone about declarant’s own current physical condition
    (b) Theory: contemporaneous statement about matter as to which declarant has unique knowledge

HYPO 80. Plaintiff, whose arm was broken in accident with Defendant, sues for damages for pain and suffering. Plaintiff may, of course, testify about the pain she experienced. But Plaintiff also calls Neighbor to testify, (a) “I was with Plaintiff last July when she said, ‘I’m feeling a lot of pain in my arm’ and again in December when she said (b) ‘I sure did feel a lot of pain in my arm last July.’” Admissible over hearsay objections?
• (a): ADMISSIBLE bc it’s a statement of then-existing contemporaneous physical pain
• (b): INADMISSIBLE bc the statement was not about Plaitiff’s current physical condition when the statement was made. Plaintiff’s statement, at the time it was made, was about past physical condition.

40
Q

VI. HEARSAY

F. Hearsay Exceptions

  1. Statement for purpose of medical treatment or diagnosis
A
  1. Statement for purpose of medical treatment or diagnosis
    (a) Statement made to anyone (but usually involves medical personnel) concerning past or present symptoms or general cause of condition for the purpose of treatment or diagnosis
    (b) Theory: motive to be honest and accurate to get good medical assessment
HYPO 81. Plaintiff v. Defendant for pain-and-suffering damages based on alleged accident at Defendant’s store. At trial, Plaintiff calls one of her treating physicians to testify, “When Plaintiff came to see me for treatment a year after the accident, she said, (a) ‘The pain in my arm is killing me. (b) I’ve been losing sleep at night for the past six months because of the pain in my arm. (c) This all started when I fell down the stairway— (d) the one with no treads at Defendant’s store.” Admissible over hearsay objections? 
•	(a): ADMISSIBLE
•	(b): ADMISSIBLE
•	(c): ADMISSIBLE
•	(d): INADMISSIBLE

Note: Statements (a), (b) and (c) would also be admissible if Plaintiff was speaking to a physician who was retained solely for the purpose of testifying as an expert witness. “Medical diagnosis” includes diagnosis for the purpose of giving an expert opinion.

41
Q

VI. HEARSAY

F. Hearsay Exceptions

  1. Business Records
A
  1. Business Records
    (a) Elements:

(1) Records of any type of business
(2) made in the regular course of the business (germane to business)
(3) the business regularly keeps such records
(4) made at or about the time of the event recorded
(5) contents consist of—
information observed by employees of the business, or
a statement that falls within an independent hearsay exception

(b) Theory: Businesses depend on accurate, up-to-date record-keeping, and accuracy is likely when employees are under a business duty to make such records. Useful as substitute for in-court testimony of employees.

Proving Business Records Foundation:

(1) Call sponsoring witness to testify to the 5 elements of business records hearsay exception; witness need not be author of report—can be records custodian or any other knowledgeable person within the business, or
(2) Written certification under oath attesting to elements of business records hearsay exception
(3) Texas: call sponsoring witness or use “self-authentication” affidavit procedure

42
Q

VI. HEARSAY

F. Hearsay Exceptions

  1. Public Records
A
  1. Public Records
    (a) Records of a public office or agency setting forth:
    (1) the activities of the office or agency (e.g., payroll records); or
    (2) matters observed pursuant to a duty imposed by law (e.g., Weather Bureau records of temperature); or
    (3) findings of fact or opinion resulting from an investigation authorized by law (e.g., FAA report on cause of plane crash).
    (b) Exception: Police reports and investigatory findings are not admissible against the defendant in a criminal case. Nor is the prosecution in such cases allowed to introduce a police report against the defendant under the alternative theory of business records.
43
Q

VI. HEARSAY

F. Hearsay Exceptions

  1. Impeachment of Hearsay Declarants
A

Opponent may use any of the impeachment methods to attack the credibility of a hearsay declarant.

HYPO 82. Shooter is on trial for murder of Victim. In hospital bed, Victim told the nurse, “I’m feeling pretty good considering Billy Ray tried to kill me.” The next day, Victim told a visitor, “I know I’m about to die. Shooter’s the one who shot me.” Prosecution introduces Victim’s statement to the visitor as a dying declaration. Should Shooter be allowed to introduce Victim’s statement to the nurse?
• Victims 2nd statement to Nurse is dying declaration.
• 1st statement to nurse doesn’t meet dying declaration, BUT bc the declarant’s credibility is at issue once the dying declaration comes in and the 1st statement is a prior inconsistent statement, the 1st statement is ADMISSIBLE to impeach/attack declarant’s credibility.